Bạn chưa đăng nhập. Vui lòng đăng nhập để hỏi bài
Nơi gió về

Cho \(a,b\inℕ^∗\)thỏa mãn \(\frac{ab+1}{a+b}< \frac{3}{2}\). Tìm Max \(P=\frac{a^3b^3+1}{a^3+b^3}\)

P/S: giả thiết ko liên quan càng tốt )

Pham Quoc Cuong
8 tháng 5 2018 lúc 22:58

Ko mất tính tổng quát, giả sử \(a\le b\)

+) Với a= 1

\(\Rightarrow\frac{b+1}{b+1}< \frac{3}{2}\Rightarrow1< \frac{3}{2}\left(TM\right)\)

Khi đó \(P=\frac{b^3+1}{b^3+1}=1\)

+) Với a=2

\(\Rightarrow\frac{2b+1}{b+2}< \frac{3}{2}\Leftrightarrow b< 4\)\(b\ge a=2\Rightarrow b\in\left\{2;3\right\}\) 

* Khi b=2 \(\Rightarrow A=\frac{65}{16}\)

* Khi b=3 \(\Rightarrow A=\frac{31}{5}\)

+) Với \(a\ge3\)

\(\Rightarrow\frac{ab+1}{a+b}\ge\frac{3b+1}{2b}>\frac{3}{2}\left(KTM\right)\)

Vậy ...


Các câu hỏi tương tự
Nguyễn Thiều Công Thành
Xem chi tiết
Nguyễn Thiều Công Thành
Xem chi tiết
Dương Thu Ngọc
Xem chi tiết
Thanh Tùng DZ
Xem chi tiết
Quyết Tâm Chiến Thắng
Xem chi tiết
Phạm Thị Huyền Trang
Xem chi tiết
Nhữ Khánh Linh
Xem chi tiết
Quyết Tâm Chiến Thắng
Xem chi tiết
Văn thành
Xem chi tiết